A four line classified ad costs 25 for a week. Each additional line costs 2. What is the weekly cost for a 12 line ad

Answers

Answer 1

Answer: 41

Step-by-step explanation:

If each line costs 2, then for the four-line ad, the cost is the initial price plus 4(2), meaning the initial cost is 25-8=17.

Thus, the cost of a 12-line ad is 17+2(12)=41


Related Questions

What type of relationship is expressed below?
5x = 2x 9
O equation
O function
O inequality

Answers

Equation. A function has at least 2 variables: an output variable and one or more input variables. An equation states that two expressions are equal, and it may involve any number of variables. Inequality is more of < > equation.

Write the algebraic expression for the following statement.

The sum of a number and 3 is 14.

Answers

Answer:

the algebraic expression is; x+3=14

Find the x-intercept of the rational function.

A (-5,0)
B (0,-5)
C (0, -1)
D (-1, 0)

Answers

Answer:

(-5,0)

Step-by-step explanation:

The x intercept is where it crosses the x axis

It crosses the x axis where x = -5

The y value is 0

(-5,0)

Answer:

A  

Step-by-step explanation:

The x -intercept is 'shorthand' for  x - AXIS intercept

  where the graph crosses the x - axis

      this will always have a   ZERO  value for the 'y'- coordinate

looking at the graph this point is   -5,0

In triangle PQR, find the measure of angle Q.
P= 64 degrees
Q= X
R= 2x

Answers

Answer:

38.667°

Step-by-step explanation:

the total sum of 3 angles in a triangle is 180

in triangle PQR this means:

P+Q+R = 180

so:

64+x+2x=180

64+3x=180

3x=116

x=116/3=38.667

So Q=X=38.667

What is the transformation of C(9, 3) when dilated with a scale factor of 1/3,
using the point (3, 6) as the center of dilation

Answers

The transformation of C(9, 3) when dilated with a scale factor of 1/3, using the point (3, 6) as the center of dilation would be an option B: C'(3,1).

What is Dilation transformation?

A dilation transformation is a transformation that changes the size of the original figure but the shape remains unchanged.

If any figure is dilated by a scale factor k with the center of dilation as the origin.

Then the change of transformation in each of the vertices of the figure is given:

(x,y) → (kx, ky)

It is given a point C which is located at C(9,3).

Hence, here k=3

We get:

C(9,3) → C'(9×3,3×3)

C(9,3) → C'(27,9) = C'(3,1)

Hence, the transformation of C(9, 3) when dilated with a scale factor of 1/3, using the point (3, 6) as the center of dilation would be an option B: C'(3,1).

Learn more about dilation;

https://brainly.com/question/6639345

#SPJ1

Answer:

(5,5)

Step-by-step explanation:

count from the center

8) Mark ran a mean distance of 13.2 km in five days. The next day, Mark ran 20 km.
Find the mean distance Mark ran in the six days.

Answers

Answer:

16.6

Step-by-step explanation:

Add the numbers together.

13.2+20=33.2

Now divide the number by 2.

33.2/2=16.6

Hope this helps!

If not, I am sorry.

please place the dot for me(20 points will give brainliest!!!)

Answers

Answer:

2,

Step-by-step explanation:

The vertex is the very highest or very lowest point. X coordinates are listed first, then Y.

2.865; cut it to the ten thousandths

Answers

Answer:

2.865.

Step-by-step explanation:

first get to know the place value of each digit is you move on you will realise that there's no digit for tens of thousands that means that you can add a zero which which will be counted as tens of thousands and since it's not past 5 it can't change the other number. I mean it can't add 1 to 5 so the number remains 2.865 good day.

Elaine saw statistics showing that union employees make an average of 30 percent more than nonunion workers, so when she was looking for a new job, she looked for one at a company that was unionized. Elaine was motivated by

Answers

Elaine saw statistics showing that union employees make an average of 30 percent more than nonunion workers, so when she was looking for a new job, she looked for one at a company that was unionized. Elaine was motivated by Economic needs.

What do we mean by economic needs?

By economic need Elaine is trying to get a company that would pay her more. Hence the need to go for the Unionized company.

This is given that the Unionized company is said to earn more than the other company. Hence her motivation is due to her economic needs.

Read more on economic needs here:

https://brainly.com/question/17889008

#SPJ1

Identify the equation with an x-intercept of 3 and a y-intercept of -2.

Answers

The equation of line is 2x  - 3y = 6.

What is intercept?

The x-intercept is the point where a line crosses the x-axis, and the y-intercept is the point where a line crosses the y-axis.

The x-intercept is 3 and the y-intercept is -2.

So, the line passes through the points  (3, 0) and (0, -2)

Now, slope of line is

m= -2-0/ 0-3 = 2/3

Equation of line is

y = 2/3 x - 2

y = 2x-6 /3

3y= 2x -6

2x  - 3y = 6

Learn more about intercept here:

https://brainly.com/question/14180189

#SPJ1

what is 2(5+34-67/45*43)^2 equal to?

Answers

The value of the give expression is  [tex]\frac{313}{748845}[/tex] OR 4.18 × 10⁻⁴

Evaluating an Expression

From the question, we are to determine the value of

2(5+34-67/45*43)^2

This can be written as

[tex]2 (\frac{5+34-67}{45 \times 43 })^{2}[/tex]

First, we will simplify the bracket

[tex]2 (\frac{-28}{1935 })^{2}[/tex]

Then, we get

[tex]2 \times \frac{-28}{1935 }\times \frac{-28}{1935 }[/tex]

= [tex]\frac{1568}{3744225}[/tex]

= [tex]\frac{313}{748845}[/tex] OR 4.18 × 10⁻⁴

Hence, the value of the give expression is  [tex]\frac{313}{748845}[/tex] OR 4.18 × 10⁻⁴

Lear more on Evaluating an expression here: https://brainly.com/question/23133222

#SPJ1

Simplify 10√3x +4√3x +5√3x

Answers

Add the like terms :

10√3x + 4√3x + 5√3x

(10 + 4 + 5)√3x

19√3x

X^2 + y^2 = 81 write down the equations of the two horizontal lines that are tangent to c

Answers

Answer:

Step-by-step explanation:

x² + y² = 81

x + y = √81

x + y = 9

x ≥ 9

y ≤ 9

Choose any set of Pythagorean triplets of your choice and illustrate them using graph sheet.
Example given below:

Answers

The required Pythagorean triplets are hypotenuse, perpendicular and base is 13, 12 and 5 respectively.

What are Pythagorean triplets?  

In a right angled triangle, its side, such as hypotenuse, perpendicular and base  is Pythagorean triplets.

We have

Hypohypotenuse = 13
perpendicular = 12
base = 5
Which satisfies the condition of the Pythagorean theorem.

[tex]a^2+b^2=c^2\\12^2+5^2=c^2\\169=c^2[/tex]

and c = 13

Thus, the required value of Pythagorean triplets satisfies the Pythagorean theorem.  

Learn  more about Pythagorean triplets here:
https://brainly.com/question/22160915

#SPJ1
 

 

The radius of a circle is 7 metres. What is the length of a 135° arc?

Answers

Answer:

16.5 meters

Step-by-step explanation:

The length of an arc of a circle = 2πr(C/360°) ;where:

C is the central angle of the arc,

R is the radius of the arc

Length of the arc = 2π(7)(135/360)

= 16.493

= 16.5 metres (approx.)

Answer:

Arc Length: 5.25π or [tex]\frac{21}{4} \pi[/tex]

Explanation:

An arc is the part of the circumference of a circle.

Formula:

[tex]\sf Arc \ Length = \dfrac{\theta}{360} \ \times \ 2\pi \ \times radius[/tex]

If applied:

[tex]\sf Arc \ Length = \dfrac{135}{360} \ \times \ 2\pi \ \times 7[/tex]

[tex]\sf Arc \ Length = \dfrac{3}{8} \ \times \ 14\pi[/tex]

[tex]\sf Arc \ Length = \dfrac{21}{4} \pi[/tex]

[tex]\sf Arc \ Length = 5.25 \pi[/tex]

the graph of F(x) can be stretched vertically and flipped over the x axis to produce the graph of G(x) if F(x)=x^2 which of the following could be the equation of G(x)

A. G(x)=-1/5x^2
B. G(x)=-5x^2
C. G(x)=5x^2
D. G(x)=1/5x^2

Answers

Answer:

g(x) = -5x²

(option B)

Step-by-step explanation:

we know that our original graph, f(x) = x² is a parabola.

So, we can consider what happens when we adjust the function/equation of a parabola.

when we "vertically stretch" a parabola, we are increasing the value of x.

 think of it this way: the steepness of a slope is rise over run. If we rise ten, and run one, that's going be a lot more steep than if we rise 1, run 1.

Let's say our x = 5

if f(x)=x²

f(5) = 25

> y value / steepness is 25

f(x) = 3x²

f(5) = 75

 > y value / steepness is 75

So, we are looking for an equation with an increase in x present.

When a parabola has been flipped over the x-axis, we know that the original equation now includes a negative

suppose that x = 1

if y = x² ; y = 1² = 1

if y = -(x²) ; y = -(1²) ; y = -1

So, when we set x to be negative, we make our y-values end up as negative also (which makes the graph look as if it has been flipped upside-down)

This means that we are looking for a function with a negative x value.

So, we are looking for a negative x-value that is multiplied by a number >1

The graph that fits our requirements is g(x) = -5x²

hope this helps!!

factoris fully 2ab² + 8a²b​

Answers

Answer:

2ab(b + 4a​)

Step-by-step explanation:

2ab² + 8a²b​

Factor Out the 2:

2(ab² + 4a²b​)

Factor Out the a:

2a(b² + 4ab​)

Factor Out the b:

2ab(b + 4a​)

In the equation 17x2 = 12x, the value of c is:
17
12
0

Answers

Answer: 0

Detailed Explanation:

General Form of a Quadratic Equation :-

=> ax^2 + bx + c = 0

a = Coefficient of x^2
b = Coefficient of x
c = Constant Term

In the Given Equation :-

17x^2 = 12x
17x^2 - 12x = 0
=> 17x^2 - 12x + 0 = 0

Therefore,
a = 17
b = -12
c = 0

Hence, value of ‘c’ = 0

Quick please!!! Buford, the farm hand, is preparing feed for
the animals. He wants to mix the contents of a
bag containing 50% corn with one containing
90% corn to optain a 30-pound bag containing
80% corn. How many pounds of the 50% bag
should he use? (write your answer as a
decimal.)

Answers

Buford should use 7.5 pounds of the 50% corn bag.

How do we solve a system of linear equations from word problems?

A system of linear equation is an equation containing two different equations with variables and the coefficient.

From the given information, let:

The weight of the bag containing 50% corn be = xThe weight of the bag containing 90% corn be = y

Sp, we can a system of a linear equation that says:

x + y = 30

0.5x + 0.9y = 0.8(30)

Using elimination method

x + y = 30

-

x - 1.8y = -48

0 - 0.8y = -18

y = 18/0.8

y = 22.5

From x + y = 30

x = 30 - 22.5

x = 7.5

Therefore, we can conclude that he should use 7.5 pounds of the 50% corn bag.

Learn more about the system of the linear equations here:

https://brainly.com/question/2226590

#SPJ1

11. Find the amount of interest on $600, if a bank is paying 5.5% interest.
[A] $30 [B] $3.30
[C] $330
[D] $33
[E] $23

Answers

B) 3.30 Since 600/100= 6 x 5.5= 3.30
B 3.30 is the answer

experimental verification​

Answers

See below for the verification of ∠ACD =  ∠ABC + ∠BAC

How to verify the angles?

The following proof works for both triangles.

The sum of angles in a triangle is 180 degrees.

So, we have:

∠ABC + ∠BAC + ∠ACB = 180

Make ∠ACB the subject

∠ACB = 180 - ∠ABC - ∠BAC

The sum of angles on a straight line is 180 degrees.

So, we have:

∠ACD + ∠ACB = 180

Make ∠ACB the subject

∠ACB = 180 - ∠ACD

Substitute ∠ACB = 180 - ∠ACD in ∠ACB = 180 - ∠ABC - ∠BAC

180 - ∠ACD = 180 - ∠ABC - ∠BAC

Subtract 180 from both sides

- ∠ACD = - ∠ABC - ∠BAC

Multiply both sides by -1

∠ACD =  ∠ABC + ∠BAC

Hence, ∠ACD =  ∠ABC + ∠BAC has been verified

Read more about proofs at:

https://brainly.com/question/24839702

#SPJ1

Figure 1 is similar to figure 2. What is TS?
1 unit
4 units
8 units
2 units​

Answers

Answer:

8 units

Step-by-step explanation:

ratios  

(x-3) /( x+1)  = (x-2)/(x+3)     cross multiply

x^2 -x -2 = x^2 - 9

  -x - 2 + 9 = 0

    x = 7          

           so TS = x + 1 = 8 units

Probability of Next Candy
Fill in the chart below with the probability of choosing the next candy.

Answers

Answer:

8 is correct.

Mark as brainlist, thanks me, and rate. Hope it helps you!!!

What is the factored form of the following expressions?

x2 – x – 42

Answers

Answer:

(x-7)(x+6)

Step-by-step explanation:

2x * -42= -84x
So you find what is common between the two then fix it

PLEASE ANSWER ASAP!!!!!11
The nth term of a sequence is represented by 2n^3+25n^2+32n−15/ 6n^4+2n^3−11n^2−2n+17.

What is the limit of the the nth term as x becomes increasingly large?

A. 0
B. 1/3
C. 3
D. The limit does not exist

Answers

The limit of the nth term of a sequence does not exist option (D) is correct.

What is the limit?

A limit is a value at which a function approaches the output for the given values in mathematics. Limits are used to determine integrals, derivatives, and continuity in calculus and mathematics.

We have an expression:

[tex]= \rm \dfrac{2n^5+25n^2+32n-15}{6n^4+2n^3-11n^2-2n+17}[/tex]

Applying limit:

[tex]= \lim_{n \to \infty} \rm \dfrac{2n^5+25n^2+32n-15}{6n^4+2n^3-11n^2-2n+17}[/tex]

[tex]=\lim _{n\to \infty \:} n \left(\dfrac{2+\dfrac{25}{n^3}+\dfrac{32}{n^4}-\dfrac{15}{n^5}}{6+\dfrac{2}{n}-\dfrac{11}{n^2}-\dfrac{2}{n^3}+\dfrac{17}{n^4}}\right)[/tex]

= ∞ = The limit does not exist

Thus, the limit of the nth term of a sequence does not exist option (D) is correct.

Learn more about the limit here:

brainly.com/question/8533149

#SPJ1

If two angles have the same angle measure, then they are said to be____.

A. Complementary
B.adjacent
C.congruent

Answers

Answer:

C

Step-by-step explanation:

2 equal angles are said to be congruent

Congruent angles have the same measure is answer is C

What is the slope-intercept equation of the line below?

Answers

Answer:

y= 2x - 3

Step-by-step explanation:

To calculate the slope (or gradient) of a line, take two points on the line whose x and y values are easy to read. Then divide the difference of they y-coordinate values by the difference of their x-coordinate values.

Here I have taken the points p (3, 3) and q (1, -1) to calculate the slope.

• slope = [tex]\frac{y_{2} - y_{1} }{x_{2} - x_{1}}[/tex]

         = [tex]\frac{3 - (-1)}{3 - 1}[/tex]

         = [tex]\frac{4}{2}[/tex]

         = 2  

The intercept is the y-coordinate value of the point at which the line crosses ("intercepts") the y-axis. I've marked the intercept point with a green line.

• intercept = -3

The slope-intercept equation of a line takes the form:

y = mx + c

where 'm' is the slope and 'c' is the y-intercept.

Substituting the values we found into the equation gives us:

y = 2x + (-3)

y = 2x - 3

find the equation of the arithmetic sequence in which t(5)=40 and t(10)=70.

Answers

The equation of the arithmetic sequence in which  t(5)=40 and t(10)=70 is tₙ = 10 + 6n

How to find the equation of an arithmetic sequence?

The equation of an arithmetic sequence can be represented as follows;

tₙ = a + (n  -1)d

where

a = first termd = common differencen = number of terms

Hence,

70 = a + 9d

40 = a  + 4d

30 = 5d

d = 30 / 5

d = 6

Therefore,

40 = a + 4(6)

40 - 24 = a

a = 16

Hence, the equation is as follows:

tₙ = 16 + (n - 1)6

tₙ = 16 + 6n - 6

tₙ = 16 - 6 + 6n

tₙ = 10 + 6n

learn more on arithmetic sequence here: https://brainly.com/question/18269451

#SPJ1

Write a system of equations that could be used to solve the situation described below.

You find 12 coins under the couch. Every coin is either a nickel or a penny, and they add up to a total of 32 cents. How many of each type of coin do you have? Let x represent the number of pennies and y represent the number of nickels.

Please select the best answer from the choices provided

A. x+y=12
0.05x+0.01y=0.32

B. x+y=0.32
0.01x+0.05y=12

C. x+y=12
0.01x+0.05y=0.32

D. not enough information

Answers

If there are 12 coins under the couch and every coin is either a nickel or a penny and together they are forming 32 cents then the equations which represent the problem will be x+y=12, 0.01x+0.05y=0.32. The correct option is B which is x+y=12, 0.01x+0.05y=0.32.

Given There are 12 coins and total money is 32 cents.

Let the number of pennies be x and the number of  nickels be y.

According to question there are 12 coins so the first equation becomes:

x+y=12.

Then we have been told that together they amount to 32 cents.

We know that 1 penny is 1 cent coin and 1 nickel is 5 cent coin so the equation becomes :

1*x+5*x=32

x+5y=32

converting into dollar

0.01x+0.05y=0.32.

Hence the right equations showing the problem of nickel and pennies are x+y=12, 0.01x+0.05y=0.32.

Learn more about equations here https://brainly.com/question/2972832

#SPJ10

Mara found the length of time of an investment. The principal of the investment was $4,300, the interest rate was 6.2 percent, and the interest was $2,666. Mara made an error in her work.

I = p r t. 2666 = (4300) (0.062) t. 2666 = (266.6) t. StartFraction 266.6 over 2666 EndFraction = t. 0.1 = t.

What was Mara’s error?
Mara did not substitute the values from the problem into the formula correctly.
Mara did not multiply correctly.
Mara did not divide correctly.
Mara divided when she should have multiplied.

Answers

The error that Maria made is that she did not divide correctly.

What error did Maria make?

The simple interest that is paid on an amount of money is a function of the amount deposited, time and interest rate.

Simple interest = principal x time x interest rate

2666 = 4300 x 0.062 x t

2666 = 266.60t

t = 2666 /  266.60

t = 10 years

To learn more about simple interest, please check: brainly.com/question/27328409

#SPJ1

Other Questions
Which of the following statements about psychological constructs is true?A.Psychological constructs are easy to define and measure.B.Psychological constructs represent concepts that cannot be observed or measured directly.C.Psychological constructs cannot be researched in psychology because they are unobservable.D.Psychological constructs represent measurable overt behavior.Please select the best answer from the choices provided The table below shows the results of a survey that asked 1052 adults from a certain country if they favored or opposed a tax to fund education. A person is selected atrandom. Complete parts (a) through (c).(a) Find the probability that the person opposed the tax or is female.P(opposed the tax or is female) =(Round to the nearest thousandth as needed.)(b) Find the probability that the person supports the tax or is male.P(supports the tax or is male) = (Round to the nearest thousandth as needed.)(c) Find the probability that the person is not unsure or is female.P(is not unsure or is female)= (Round to the nearest thousandth as needed.) Neither n2 nor o2 are greenhouse gases because GOD PLEASE HELP!Which of these is NOT a key factor that contributed to the financial crisis that started in 2007?bond rating agencies giving CDOs very low ratings, discouraging investors to investhighly leveraged investment accounts that magnified losses when bond prices decreasedexcessive use of subprime mortgages to borrowers with poor creditbanks being incentivized to sell risky mortgages as mortgage-backed securities Is the point (11, 15) on the circle defined by (x-9) + (y-15) = 4?O YesO No What was the purpose of the Munich conference? Which is true regarding the graphed function f(x)? f(O) = 3 f(5) = - 1 f(3) = 2O f(2) = -2 If a utility burned 7.57 million tons of coal that was 2.00% sulfur by weight, how many tons of sulfur dioxide were emitted? Answer in scientific notation. find x if 2+x = -2/3 Marion is going to the hospital for a triple bypass operation. she will have general anesthesia, an intravenous catheter, surgical wounds, and a urinary catheter. which healthcare-associated infection is she at greatest risk for contracting Y W P 8 cm E R WERP is a square. Find.1. m2WER= 2. measure of PE =_______ 3. measure of WA = 4. Value of y= Where should your passengers be while the boat is tied to the dock for fueling?. if the unit rate is 30 miles per hour , then how much miles is traveled in 9 hours Examine the following three premises: i. All kids who have a smartphone play mobile legends. ii. All people who play mobile legends enjoy life. iii. Some college instructors enjoy life. Now consider each of the following six conclusions. For each conclusion, determine whether the argument formed by the three premises and the conclusion is valid or invalid. 1. Therefore, some college instructors have a smartphone. 2. Therefore, some college instructors play mobile legends. 3. Therefore, some kids who play mobile legends are college instructors. Examine the following three premises : i . All kids who have a smartphone play mobile legends . ii . All people who play mobile legends enjoy life . iii . Some college instructors enjoy life . Now consider each of the following six conclusions . For each conclusion , determine whether the argument formed by the three premises and the conclusion is valid or invalid . 1. Therefore , some college instructors have a smartphone . 2. Therefore , some college instructors play mobile legends . 3. Therefore , some kids who play mobile legends are college instructors . Complete the steps to factor the polynomial by grouping. p(x) = x3 5x2 x 5 p(x) = x2 (x ) (x 5) p(x) = (x2 )(x 5) p(x) = (x )(x 1)(x ) The __________ is also known as the Empty Quarter because of its harsh sand dunes and uninhabited areas.A.Shatt al-ArabB.Bab al-MandabC.Dasht-e KavirD.Rub al-Khali What are your reactions to the New York Times short film? What in the film stood out to you? What did you learn? Were there any surprises? Anything that challenged what you know or thought you knew? What questions do you still have? Rubric In 2010, your company paid overtime wages or hired temporary help during 32 weeks of the year. Overtime was paid for 26 weeks and temporary help was hired for 15 weeks. If at years end an auditor checks your accounting records and randomly selects one week to check the companys payroll, what is the probability that the auditor will select a week in which you paid overtime wages and hired temporary help? Given the two similar triangles below, which proportion is not true? I need help lol xoxoxo